Download as pdf or txt
Download as pdf or txt
You are on page 1of 33

UPSC 2024

PRACTICE
BOOKLET
UNIT-1 ( NUMBER SYSTEM )

Visit Our Website


www.iassetu.com

www.iassetu.com 1 +91 8826619699


INDEX
TOPIC PAGE NO.
Questions 2-10
Answer Key 11
Explanation 12-31
Rough Page 32
1. Find the remainder when 399 in one way only.
isdivided by 28: II. It can be written as sum of the
a) 3 cubes of two positive integers
in two ways only.
b) 9 Which of the statements
c) 27 given aboveis/are correct?
d) 1

2. A Monkey climbs a pole of 75meter.


After climbing 3 meters in a minute, a) 1 only
he slips down 1 meter in the next
minute. In how many minutes the b) 2 only
monkey will reach the top of the pole. c) Both 1 and 2
a) 72 d) Neither 1 nor 2
b) 73
c) 74 7. What is the remainder when 157 +197
d) 75 is divided by 34?
a) 0
3. Consider the following sum: b) 1
P+2P+P5+4P+P7= 1Q4 c) 2
In the given sum, find the value of P d) 3
+ Q.
a) 2 8. If n, (n+2) and (n+4) are three odd
numbers and prime numbers both,
b) 10
then what is the number of possible
c) 4 solutions for n?
d) 7 a) 2
b) 4
4. What is the sum of first 43 c) 1
oddnumbers? d) 3
a) 1848
b) 1849
9. Consider the following statements:
c) 1850 I. 324918 is divisible by 9.
d) 1851 II. 191 is a prime number.
Which of the statements given
aboveis/are correct?
5. What is the units digit in
theexpression 792? a) 1 only
a) 1 b) 2 only
b) 3 c) Both 1 and 2
c) 6 d) Neither 1 nor 2
d) 7
10. The sum of the five consecutive odd
numbers is 195. What is the
6. Consider the following statements
smallest number among them?
regarding the number 1729.
a) 33
I. It can be written as sum of the
cubes of two positive integers b) 35

www.iassetu.com 2 +91 8826619699


c) 37 c) 20
d) 22
d) 39
15. Integers are listed from 300 to 500. In
11. The product of two numbers is 5760 how many integers is the sum of the
and their HCF is 12. How much such digits 7?
pair of numbers can be formed? a) 6
a) 1
b) 7
b) 2
c) 8
c) 3
d) 9
d) 4
16. How many zeros are there at
theend of 40!?
a) 9
b) 11
12. The maximum number of students c) 13
among whom 897 pens and 690
d) 10
pencils can be distributed in such a
way that each students gets same
number of pens and same number of 17. Find the number of integers from
pencils is: 400 to 600 that are not divisible by
a) 690 either 3 or 5.
b) 1587 a) 109

c) 897 b) 112

d) 69 c) 110
d) 111
13. If A and B are different integers both
divisible by 9 and A > B, then 18. Two numbers, when divided by 19,
consider the following statements. leaves remainder 13 and 7
I. A-B is divisible by 9. respectively. If the sum of those two
numbers is divided by 19, the
II. A+B is divisible by 18. remainder will be:
III. A2+B2 is divisible by 81. a) 1
Which of the statements given above are
b) 3
correct?
a) 1 and 2 only c) 5
b) 2 and 3 only d) 7
c) 1 and 3 only
d) 1, 2 and 3 19. What is the greatest 4-digit number
such that when it is divided by 12,
16, 24 and 32 leaves a remainder of
6?
14. There are eleven two-digit a) 9976
consecutive even numbers. If the
b) 9982
mean of the first five such numbers is
16, then what is the mean of all the c) 9984
eleven numbers? d) 9990
a) 24
b) 18
20. Consider the following statements

www.iassetu.com 3 +91 8826619699


in respect of two natural numbers p correct, then the correct answer will
and q such that p is an Even number be:
and q is an odd number: a) 658281
I. p x q is an odd number. b) 658283
II. pq is an even number. c) 658288
III. qp is an odd number. d) 658287
Which of the above statements givenabove
are correct? 25. If n is a whole number greater than
1 then n2(n2-1) is always divisible
a) 1 and 2 only by:
b) 2 and 3 only a) 3
c) 1 and 3 only b) 6
d) 1, 2 and 3 c) 9
d) 12
21. A positive number, when increased
by 5, equals 50 times its reciprocal.
Find the number. 26. The expression 74n – 54n where n is an
even number is always divisible by
a) 5 which of the following number?
b) 10 I. 2
c) 15 II. 23
d) 20 III. 111
IV. 37
22. Find the sum to 100 terms of the Select the correct answer using the
series 1 +3 +4 +5 +7 +7 +10 +9 + codegiven below:
......
a) Only 1
a) 6300
b) Only 2
b) 6325
c) 1 and 3 only
c) 6350
d) 1 ,3 and 4 only
d) 6375

27. Two Statements S1 and S2 are


givenbelow with regard to two
23. Let A2BC and DE3F be four-digit numbers followed by a
numbers where each letter represents Question:
a different digit greater than 3. If the Statements:
sum of the numbers is 16694, then S1: Their
what is the difference between the product is
values of A and D? 45.
a) 3 S2: Their
sum is 14.
b) 4
c) 2 Question: What are the two numbers?
d) 1 Which one of the following is
correct inrespect of the above
Statements and the Question?
24. A boy multiplies 903 by a certain
number and obtains 659297 as the
answer. If in the answer, both 9’s a) S1 alone is sufficient to
are wrong but the other digits are answer theQuestion.

www.iassetu.com 4 +91 8826619699


b) S2 alone is sufficient to a) 2
answer theQuestion. b) 3
c) S1 and S2 together are
sufficient toanswer the c) 4
Question. d) 5
d) S1 and S2 together
are notsufficient to 33. When a certain two-digit number is
answer the Question. subtracted to another two-digit
number having the same digits in
28. If a positive integer leaves reverse order, then their difference is
remainder 26 when divided by a perfect square. How many such
143, then what is the remainder two-digit numbers are there?
obtained on dividing the same
number by 11? a) 4
a) 4 b) 6
b) 6 c) 8
c) 7 d) 5
d) 2
34. A appeared for an exam. He was given
160 problems to solve. He tried to
29. If the number 756524P57Q is solve all of them correctly but some
divisible by 72, then find the of them went wrong. Any how he
value of P and Q. scored 100. His score was calculated
a) 4 and 6 by subtracting 2 times the number of
b) 6 and 8 wrong answers from the number of
correct answers. Can you tell how
c) 7 and 6 many problems he solved correctly?
d) 2 and 4 a) 110
b) 120
30. The product of two numbers is 9216 c) 130
and the quotient, when the larger one d) 140
is divided by the smaller, is 16. The
sum of the numbers is: 35. In a school, every student is assigned
a) 400 a unique identification number. A
b) 384 student is a basketballplayer if and only
if the identification number is divisible
c) 380 by 3, whereas a student is cricketer if
d) 376 and only if the identification number is
31. What is the remainder when 7100 is divisible by 5. If every number from 1 to
divided by 101? 100 is assigned to a student, then how
many of them play cricket as well as
a) 1
basketball?
b) 3
a) 5
c) 5
b) 8
d) 7
c) 7
d) 6
32. Following expression holds true if
we replace some of ‘+’ signs by ‘×’
signs. 36. In how many ways can you express 972
as a product of two of its factors?
1 + 2 + 3 + 4 + 5 + 6 + 7 + 8 + 9 + 10 = 156.
How many ‘+’ signs are needed to be replaced a) 8
by ‘×’? b) 7

www.iassetu.com 5 +91 8826619699


c) 10 a) 5(a + b)
d) 9 b) 6ab
c) 2 (a + 3b)
37. Find the no. of zeroes at the end ofthe d) 4a + 5b
product of 2111× 5222.
a) 111 42. Praveen has 42 romantic novels, 78 war stories,
b) 222 and 102 science books in his bookshelf. These
c) 333 books must be arranged in rows with the same
number of books and of the same type in each
d) 100
row. Then, what is the minimum number of
rows?
38. In an entrance exam, a student has to
a) 35
attempt 200 questions in 4 hours.Out of
these 200 questions, 40questions are on b) 36
QuantitativeAbility. How many minutes c) 37
he spenton Quantitative Ability section d) 33
if he spends twice as much time on each
Quantitative Ability problem asspent for
43. Consider the following statements in
every other question?
respect of two naturalnumbers p and q
a) 40 such that p is aprime number and q is a
b) 60 compositenumber:
c) 80 I. p and q can be coprime numbers.
d) 100 II. q / p can be a prime number.
III. p + q can be an odd or an evennumber.
39. There are 22 steps in a ladder. Ainitially was at Which of the above statements are correct?
the 22nd step, and he comes down two steps a) 1 and 2 only
each time. B, initially was at 1st step and goes
one step up each time. If they start b) 2 and 3 only
simultaneously and keep their speed uniform, c) 1 and 3 only
then at which step from the bottom will they d) 1, 2 and 3
meet?
a) 7th
44.A and B each have a set number of pens. If B
b) 8th gives A ten pens, A will have three times as
c) 9th many pens as B.If A gives B ten pens, B will
d) 10th have the same number of pens as A. What is
the number of pens that A and B have
respectively?
40. It is being given that (230+ 1) is completely a) 40, 20
divisible by a whole number. Which of the
following numbers is completely divisible by b) 60, 40
this number? c) 20, 40
a) (215 + 1) d) 80, 60
b) (215 - 1) 45.For all positive even integers, a, what is the
c) (3 x 223) largest integer that is a divisor of (a + 5) (a +
7) (a + 9) (a + 11) (a + 13)?
d) (290+ 1)
a) 5
b) 25
41. If a and b are positive integers such that (5a +
c) 65
3b) is a multiple of 13, then which of the
following will alsobe divisible by 13? d) 15

www.iassetu.com 6 +91 8826619699


46.A number consists of 3 digits whose sum is 24. by 4, the sum of the productsof the first and
The middle digit is half of the sum of the other third, and of the second and fourth is 11, and
two digits and the number will be decreased the sum of the first, second and third exceeds
if its digits are reversed. The number is: the fourth by 6. What are those four positive
a) 888 numbers respectively?
b) 969 a) 4, 1, 9, 2
c) 789 b) 3, 4, 1, 2
d) 987 c) 6, 5, 4, 3
d) None of these
47. Consider the following statements:
I. There are total 25 two-digit prime numbers.
Q-51) If A is 3-digit natural number which is
II. There are seven two-digit primenumbers that
divisible by 3 and B is 2-digit even
remain primenumbers even when their digits number, then consider the following
are reversed. statements:

Which of the statements given aboveis/are i) A + B always divisible by 2.


correct?
ii) B2 always divisible by 4.
a) 1 Only iii) A× B is always divisible by 6.
b) 2 only
c) Both 1 and 2 Which of the above statement/s is/are
correct?
d) Neither 1 nor 2
A) i & ii only
48. In a three-digit number, the sum of the digits B) ii & iii only
equals the product of the digits. Then consider C) i & iii only
D) i, ii & iii
the following statements:
I. The number is always divisible by 2, 3 and 6. Q-52) What is the remainder of
91 × 93 × 94 × 96 × 97 × 98 ×99 × 100
II. The number can be either odd oreven. ?
95

Which of the statements above is/aretrue? A) 85


B) 87
a) 1 only C) 89
b) 2 only D) 92
c) Both 1 and 2 Q-53) In the expression of P23 + PP6 + 26P +
d) Neither 1 nor 2 (𝑃∗𝑃)+ 1
PPP = 6PP. Find the value of =?
2

49. At an event, 17 people shook hands with each A) 1


other before and after the meeting. How B) 4
many total numbers of handshakes occurred? C) 9
a) 136 D) 16
b) 274
Q-54) 6548 what is the unit digit number in this
c) 272 given expression?
d) 137
A) 4
B) 6
50. There are four positive numbers in such a way C) 8
that the sum of the squares of second and third D) 9
exceedsthe sum of squares of first and fourth

www.iassetu.com 7 +91 8826619699


915324 16,17 & 18 then what is the remainder
Q-55) Find the Remainder of ?
15 respectively?
A) 1 A) 11 , 0 , 13
B) 5 B) 0 , 16 , 14
C) 9 C) 13, 16 , 19
D) 12 D) 0 , 13 , 15
Q-56) In the given expression 15 × 18 × 20 × Q-62) If the 3-digit number xyz is divisible by
22 × 25 × 95× 155× 175 × 32, find the number its own all individual digit(x,y,z), also
of zeroes? divisible by the sum all digit(x+y+z)
A) 6 and multiple of all the digits(x × y × z).
B) 8 Then find the 3-digit number xyz.
C) 5 A) 232
D) 7 B) 342
67 × 71 × 73 × 83 C) 432
Q-57) Find the Remainder of ? D) 534
64

A) 7 Q-63) 5 × 9 × 13 × 15 × 18 × 20 × 35 × 48 ×
B) 14 60 × 125 × 144 In the given expression,
C) 21 find the number of zeroes?
D) 28
A) 4
Q-58) 1,1,2,3,5,8,13,x,34,55,.... what is the B) 7
value of x ? C) 5
D) 6
A) 21
B) 23 Q-64) A, B, C & D are any natural number but
C) 25 either A & C or B & D one of the pair
D) 27 is EVEN number and another one is
ODD number then consider following
Q-59) When the natural number A,B and C is statement.
divided by 41 gives a remainder of
21,16 and 32 respectively. Find the i) (A2 + B)(C2 + D) is always even
remainder when (3A + 3B – 2C) is ii) (A + B2)(C2 + D) is always odd
divided by 41? iii) (A × C)(B × D) is always even
A) i & ii correct
A) 6 B) ii & iii correct
B) 9 C) i & iii correct
C) 12 D) i, ii & iii correct
D) 15
Q-65) If 158902 × 25910 × 5821 is divided by
Q-60) ABCD & EFGH is 4-digit natural 100 then what is the remainder?
number. Where ABCD + EFGH =
12321 and the value of A+C+E+G = x. A) 2
If the value of B + F > C + G, then B) 4
x+2
what is the value of ( 6 )3? C) 6
D) 8
A) 64
B) 125 Q-66) If the number 65892a5 is divisible by
C) 216 15 then what is the value of a?
D) 343 A) 2
Q-61) 4-digit smallest number which is B) 4
divided by 19 gives remainder of 15. If C) 6
the same 4-digit number is divided by D) 8

www.iassetu.com 8 +91 8826619699


Q-67) Consider the following expression A8 + Statement 2: No single digit even
2A + 31 + A1 = 10A. Find the value of A? number is prime number except 2.

A) 2 Statement 3: Total 16 prime numbers


B) 3 are their between 0 and 50.
C) 5
D) 7 Statement 4: Number of prime numbers
between 40 and 50 are equals to the
Q-68) If A stands for ‘-’, B stands for ‘+’, C number of prime numbers between 70
stands for ‘×’, D stands for ‘÷’ then and 80.
consider the following statement for the
expression X: Which of the statement is Not Correct?
525 A 25 B 50 C 100 D 5 A 60 A) Only 1 and 2
X= .
125 B 25 D 5 A 50 B) Only 2 and 3
C) Only 1 and 3
Statement 1: 2X + 1 is prime number.
D) Neither 1 nor 2 nor 3.
Statement 2: X2 – 2X + 1 = 172

Which of the following statement is/are Q-72) 2x2 - 38x + 68 = 0 what is the sum of all
Correct? possible value of x?

A) Only 1 A) 19
B) Only 2 B) 21
C) Both 1 and 2 C) 23
D) Neither 1 nor 2 D) 27

Q-69) Consider the following statement:


Q-73) Find number of Zeros possible at the
Statement 1: The difference between end of the given expression : 101! ×
the squares of two consecutive odd 151!
integers is always divisible by 8.
A) 35
Statement 2: The product of any 4 B) 152
consecutive even numbers is always C) 155
divisible by 384. D) 61

Which of the following statements are Q-74) A 6-digit natural Number 563P32 is
correct? divisible by 6 then what is the value of P?
A) Only 1 A) 1
B) Only 2 B) 2
C) Both 1 and 2 C) 3
D) Neither 1 nor 2 D) 4
Q-70) (3662 - 3362) is divisible by which of the Q-75) Mathematic operation coded using
following digit? symbols like:
A @ B means A + B
A) 2
A $ B means A - B
B) 3
A # B means A × B
C) 5
A & B means (A × B)2
D) 6
If the value of (3 & 5) @ 59 $ 17 # 2 = x, then
consider the following statements:
Q-71) Consider following statements:
Statement 1: x is divisible by 2, 5, 10 and 20?
5√40
Statement 1: All the single digit odd Statement 2: √x =
2
numbers are prime number except 9.

www.iassetu.com 9 +91 8826619699


Which of the following statement is/are
Correct?
A) Only 1
B) Only 2
C) Both 1 and 2
D) Neither 1 nor 2

www.iassetu.com 10 +91 8826619699


Q. No. Answer Q. No. Answer Q. No. Answer
1 c 36 d 71 C
2 b 37 a 72 A
3 b 38 c 73 D
4 b 39 b 74 B
5 a 40 d 75 B
6 b 41 d
7 a 42 c
8 c 43 d
9 c 44 a
10 b 45 d
11 b 46 d
12 d 47 d
13 c 48 b
14 d 49 c
15 d 50 b
16 a 51 B
17 c 52 A
18 a 53 A
19 d 54 B
20 b 55 A
21 a 56 B
22 b 57 A
23 c 58 A
24 d 59 A
25 d 60 A
26 d 61 A
27 c 62 C
28 a 63 B
29 c 64 B
30 a 65 A
31 a 66 B
32 b 67 A
33 d 68 C
34 d 69 C
35 d 70 B

www.iassetu.com 11 +91 8826619699


Explanation
Q. No. Answer Explanation
1 c 3 99 (33) 33 2733 2733
= = =
28 28 28 27+1

𝐚𝐧
Now We Know leaves a 𝐫𝐞𝐦𝐚𝐢𝐧𝐝𝐞𝐫 𝐨𝐟 𝐚 𝐢𝐟 𝐧 𝐢𝐬 𝐨𝐝𝐝
𝐚+𝟏
𝐚𝐧𝐝 𝟏 𝐢𝐟 𝐧 𝐢𝐬 𝐞𝐯𝐞𝐧

Here a= 27 n= 33 which is odd hence the reminder is 27.


2 b Given that the monkey climbs 3 meters in a minute.
And He slips down by 1 meter in the next minute.
Hence, the Distance covered by the monkey in 2 min = 3-1= 2 meters
He Climbs 2 meter in 2 min.
Hence, the distance covered by him in 1 min = 1 meter
Therefore, he climbs 72 meter in 72 minutes, and after climbing 72
meters, he climbs the rest remaining 3 meter in 1 minute i.e. the total 75
meter in 73 minutes.
Hence, the time taken by the monkey in climbing the pole = 73 minute
3 b Given that P+2P +P5+4P+P7= 1Q4
By Hit and Trial Method P+2P +P5+4P+P7= 1Q4.
If we take P= 1
1+21+15+41+17=95
If we take P= 2
2+22+25+42+27=118
If we take P= 3
3+23+35+43+37=141
If we take P= 4
4+24+45+44+47=164
If we take P= 5
5+25+55+45+57=187
If we take P= 6
6+26+65+46+67=210
If we take P= 7
7+27+75+47+77=233
If we take P= 8
8+28+85+48+87=286
If we take P= 9
9+29+95+49+97=279
From highlighted equation we can find that:
P = 4 and Q = 6
Therefore, P+Q = 10
Hence the correct answer is b.

www.iassetu.com 12 +91 8826619699


4 b We know that the sum of first N odd numbers = N2
Here N=43
∴ sum of first 43 odd numbers = 432= 1849
Point to Remember: sum of first n even numbers = n(n+1)
5 a For a number of the form ab if b is a multiple of 4, then
By using rule of cyclicity:
if a is an even number i.e., 2,4,6,8, then the unit’s digit is 6.
And if a is an odd number i.e., 1,3, 7,9 then unit’s digit is 1.
Here a =7 and b=92 which is multiple of 4
Hence the unit’s digit in the expression 792= 1
6 b We have
1729 = 1728 + 1 = 123 + 13
Or 1729 = 1000+729 = 103 + 93
Thus 1729 can be written as sum of the cubes of two positive integers in
two ways only.
7 a We know that (xm + ym ) is always divisible by (x + y), for all odd values of
m.
Hence 157 +197 is always divisible by 15+19=34 as here m=7.
So, the remainder here is 0.
8 c When n is a prime number then there is only one possible case that n,
n+2 and n+4 are prime numbers and odd numbers.
When n = 3 then n, n+2, n+4 = 3 , 5 ,7 which are all prime numbers and
odd.
9 c When the sum of all the digits of a number is divisible by 9 then the
number is also divisible by 9.
Here for 324918 the sum of all the digits = 3+2+4+9+1+8 = 27 which is
divisible by 9
Hence, 324918 is divisible by 9.
Therefore, statement 1 is correct.
To check if 191 is prime we check if it divisible by any prime numbers
from 2 to 13.
Since it is not divisible by 2, 3, 5, 7, 11, 13. Hence it is a prime number.
Hence statement 2 is also correct.
10 b Let the five consecutive odd numbers be x , x+2, x+4, x+6, and x+8
Then given that:
x +x+2+ x+4+ x+6+x+8 = 195
5x+20 = 195
5x= 175
x= 35
Hence option (b) is the correct answer.
Alternate Solution:
Middle term of odd term series = sum of the numbers / total terms
= 195/5

www.iassetu.com 13 +91 8826619699


= 39
So, the other term will be 35, 37, 39, 41, 43. Hence the smallest number
among them is 35.
11 b Given The product of two numbers is 5760.
As their HCF is 12.
So Let numbers be 12x and 12y.
Hence 12x × 12y = 5760
⇒ xy = 40
∴ Possible pairs are: (1, 40), (5, 8)
12 d The maximum number of students among whom 897 pens and 690
pencils can be Distributed can be obtained by finding the HCF of 897 and
690.
897= 3×13×23
690 = 3×10×23
Hence, the HCF of 897 and 690 = 3×23= 69

Hence, the maximum number of students among whom 897 pens and
690 pencils can be Distributed = 69

13 c Given that A and B are different integers both divisible by 9, then we can
write A and B as follows:
A = 9x, where x is an integer and x ≠ 0
B = 9y, where y is an integer and y ≠ 0 and x ≠ y, since A and B are
different.
Using the above expressions, let's evaluate the given statements:
A-B = 9x – 9y = 9(x-y), which is clearly divisible by 9. So, statement 1 is
correct.
A+B = 9x + 9y = 9(x+y).
Since x and y are both integers, x+y is also an integer, and thus A+B is
divisible by 9. To be divisible by 18, A and B would need to be both even,
which is not the case since they are both multiples of 9. So, statement 2
is incorrect.
A2 + B2 = (9x)2 + (9y)2 = 81(x2 + y2), which is clearly divisible by 81. So,
statement 3 is also correct.
Hence, the option (c) is the correct answer.
14 d Given that the mean of the first five two-digit even numbers is 16, so
wecan write:
(x + x+2 + x+4 + x+6 + x+8 ) / 5 = 16
Simplifying this equation, we get
5x + 20 = 80
5x = 60
x = 12

www.iassetu.com 14 +91 8826619699


Therefore, the first two-digit odd number is 12 and hence the eleven two-
digit consecutive odd numbers are: 12,14,16,18,20,22,24,26,28,30,32 and
sum of terms
Now, mean of all the eleven numbers =
number of terms

12+14+16+18+20+22+24+26+28+30+32 242
Mean term = = = 22
11 11
Hence, the correct answer is option (d).
15 d For a hundred digit of 3, we need to find two digits whose sum is 7-3=4.
There are five such pairs: (1,3), (2,2), (3,1), (4,0) and (0,4).
Each of these pairs can be combined with the hundreds digit 3 to form a
valid integer between 300 and 400 to have a digit sum of 7. Hence there
are 5 integers between 300 and 400 whose digit sum is 7, these are = 304,
313, 322, 331, 340
Now Between 400 and 500 the integers whose sum of the digits is 7= 403,
412, 421, 430
A total of 4 integers.
Hence the total integers between 300 and 500 whose sum of the digits is
7 = 5+4 = 9
16 a To find the number of zeros at the end of 40! (the factorial of 40), we
need to determine how many times 10 or 2 x 5 appears as a factor in the
prime factorization of 40!.
Since there are more factors of 2 than 5 in the prime factorization of 40!,
we only need to count the number of factors of 5.
To find the number of factors of 5 in the prime factorization of 40!, we
use :
40/5 = Quotient is 8, and 8/5 = Quotient is 1.
Hence the total number of Zeros in 40! = 8+1 =9.
Therefore, there are 9 zeros at the end of 40!.
17 c To find the number of integers from 400 to 600 that are not divisible by
either 3 or 5.
First, we find the Total numbers that are divisible by 3 or 5 between 400
and 600:
600 399 600 399 600 399
= − + + − −
3 3 5 5 15 15
= (200– 133) + (120 – 79) – (40– 26)
= 67 + 41 – 14
= 91
Now,
Total numbers between 400 and 600 = 600 – 400 + 1
= 201
∴ The number of integers from 400 to 600 that are not divisible by either
3 or 5= 201 – 91

www.iassetu.com 15 +91 8826619699


= 110
18 a We know that Dividend = Divisor × quotient + remainder
Hence the First no. = (19 × n) + 13, Let n = 1
= (19 × 1) + 13
= 32
Second No. = (19× n) + 7
= (19 × 1) + 7
= 26
According to question, sum of these two numbers = 32+26 = 58
58
Now = Remainder is 1
19
Hence The Correct answer is (a).
19 d The LCM of 12, 16, 24 and 32 = 96
9999
Now the greatest 4-digit number which is divisible by 96 is = =
96
Remainder is 15.
The greatest 4-digit number which is divisible by 96 = 9999-15 = 9984
Hence the required number = 9984+6= 9990
20 b Given that p and q are two natural numbers such that p is an Even
number and q is an odd number.
Let p= 2,4,6,8,….
q = 3, 5,7, 9,….
p x q is always an even number as Odd x Even= Even, 2x3=6, 4x7=28 etc.
Hence Statement 1 is incorrect.
Statement 2: pq such as 23 =8 , 45 = 1024 etc. is always an even number as
(Even) odd= Even.
Hence Statement 2 is correct.
Similarly, qp is an even number as (Odd) even= Odd
As 32 =9, 74= 2401 these are odd numbers.
Hence Statement 3 is correct.
21 a Let the number be x
According to question,
50
⇒x+5=
X
⇒ x2+ 5x = 50
⇒ x2 + 5x - 50 = 0
⇒ x2 + 10x - 5x - 50 = 0
⇒ x(x + 10) - 5(x + 10) = 0
⇒ (x - 5)(x + 10) = 0
⇒ x = 5 or x = -10
But x is a positive number, so x ≠ - 10
⇒x=5
∴ The required positive number is 5.
Hence Option (a) is correct answer.

www.iassetu.com 16 +91 8826619699


22 b Given, series: 1 + 3 + 4 + 5 + 7 + 7 + 10 + 9 + ......... is a combination of
two Arithmetic Progressions (AP)
S1 = 1 + 4 + 7 + 10 + ........ and S2 = 3 + 5 + 7 + 9 + ……
Now, each of the AP will have 50 terms.
Hence the Sum S = S1 + S2
n
Now the formula for the sum of n terms of the AP S1 = [ 2a + (n − 1)d]
2

Where n = no of terms, a = first term of the AP, d = common difference


50
∴S= (2 × 1 + 49 × 3) as here n = 50 , a = 1 , d =3
1
2
S1 = 25 × 149 = 3725
And S2 = 25 ×( 2×3 + 49×2)
S2 = 25×104 = 2600
Hence the sum = 3725 + 2600 = 6325
23 c Given that A2BC + DE3F = 16694
According to question,
C + F = 4, so C + F must be 14 as C and F are greater than 3 then C and F
can either be (5,9) or (6,8)
B+3+1 = 9 since 14 gives a carry 1 so B = 5.
Again,
2+E = 6 which give E=4
Now for A and D
A+D = 16. As A and D are different and greater than 3
Hence A and D must be (7 or 9)
Hence the difference between them = |9-7|
=2
Hence Option (c) is the correct answer.
24 d The given that the number is multiplied by 903 = 3 x 7 x 43.
So, the required number must be divisible by each one of 3, 7, 43.
Out of the given options and 658281 is not divisible by 7. While 658283 is
not divisible by 3.
658288 is neither divisible by 3 nor by 7.
658287 is divisible by each one of 3, 7, 43.
Hence the correct answer is (d).
25 d If n is greater than 1 than n2(n2-1) is always divisible by 12
Let n=2 greater than 1
Then we have n2(n2-1) = 22(22-1)= 4×3 = 12
If we take n= 3 then n2(n2-1) = 72
Similarly, we can show that n2(n2-1) is always divisible by 12 when n is
greater than 1.
Hence Option (d) is the correct answer.
26 d We know that an-bn is always divisible by (a-b)
Hence 74n-54n is divisible by (7-5) = 2 and 74-54= 1776
www.iassetu.com 17 +91 8826619699
Also if n is even then an-bn is divisible by ( a+b)
And for 74n-54n, 74+54= 3026
Now 1776 is divisible by 37 and 111.
Hence the given expression is divisible by 2, 37 and 111.
Hence option (d) is correct answer.
27 c Let the two numbers be x and y.
In Statement 1 Given that x × y = 45
So, numbers are: (5,9), (15,3)
According to Statement 2;
x + y = 14
So, numbers are: (5,9), (6, 8), (7, 7), (10, 4), (11, 3) (12,2) and (13,1)
Hence, the required number = (5, 9)
∴ S1 and S2 together are sufficient to answer the question, but neither S1
alone nor S2 alone is sufficient to answer the Question.
Hence Option (c) is the correct answer.
28 a Given that Positive integer = 143 × k + 26
⇒ 11 × 13× k + 11 × 2 + 4
⇒ 11(13k + 2) + 4
So, the required remainder = 4
∴ The remainder obtained on dividing the same number by 11 is 4.
Hence option (a) is correct answer.
29 c Given that:
The number 756524P57Q is divisible by 72.
Since we know that:
A number is divisible by 8 if its last 3 digit are divisible by 8.
A number is divisible by 9 if the sum of digits of the number is divisible by
9.
Given the number is divisible by 72 then the number must be divisible by
8 and 9.
Now For number 8:
⇒ 57Q must be divisible by 8
⇒ The value of Q= 6
⇒ Hence the number becomes = 756524P576
For number 9:
⇒ Sum of all digits of the given number = 7 + 5 + 6 + 5 + 2 + 4 + P + 5 + 7 +
6= 47 + P
⇒ So, 47 + P must be divisible by 9.
⇒ The value of P= 7 as 47+7= 54 is divisible by 9.
Hence P= 7 and Q= 6 is the required answer.

www.iassetu.com 18 +91 8826619699


30 a Let the numbers be x and y.
X
Given that Then, xy = 9216 and = 16
y
Xy
Then = 9216/ 16
X/y
Or y2 = 576
y = 24 and hence x= 16y = 16×24 = 384
hence the sum of numbers = x+y = 400
31 a By Fermat’s Remainder Theorem:
If P be a prime number and A be a number non-divisible by P.
AP−1
Then 𝑇ℎ𝑒 𝑅𝑒𝑚𝑎𝑖𝑛𝑑𝑒𝑟 𝑜𝑏𝑡𝑎𝑖𝑛𝑒𝑑 𝑤ℎ𝑒𝑛 = 1 if A and P are Co
P
Prime.
Hence here A = 7, P= 100, A and P are Co- Prime.
7100
Hence = Remainder is 1
101
Hence Option (a) is correct.
32 b The Given Expression:
1 + 2 + 3 + 4 + 5 + 6 + 7 + 8 + 9 + 10 = 156.
Since,1 + 2 + 3 + 4 + 5 + 6 + 7 + 8 + 9 + 10 = 55
Therefore, by replacing the signs, we need to make 101 extra.
This is possible only if we write the given expression in this way:
1 + 2×3 + 4 + 5 + 6 ×7 + 8 + 9×10 = 156
Hence three ‘+’ signs are needed to be replaced by ‘×’.
33 d As per the given in the question,
We need to find the two-digit number that is having same digits in
reverse order and their difference is a perfect square.
Now, let us assume the number = 10x + y
Then another Number in Reverse Order = 10y + x
Their difference = 10x+y-10y-x
= 9(x-y) and it is a perfect square
When x-y = 4 and
x-y = 9 but x-y = 9 only possible when x= 9 y = 0 but then the reverse
number becomes 9 which is not a 2-digit number hence we reject it.
So, When x-y =4
Using the trial-and-error method we get the following answers.
Implying that,
x = 5. y = 1
x = 6, y = 2
x = 7, y = 3
x = 8, y = 4
x=9, y =5
Therefore, the numbers are 51, 62, 73, 84, 95.
34 d Let us assume W be the number of wrong Questions and R be the number
of right Questions.

www.iassetu.com 19 +91 8826619699


Given that R - 2W = 100
R + W = 160
-3W = -60
W = 20
From, R + W = 160
R = 140
Hence Number of Questions solved correctly by A = 140
Hence option (d) is correct.
35 d Given A student is a basketball player if and only if the identification
number is divisible by 3.
And a student is cricketer if and only if the identification number is
divisible by 5.
The number of players who play cricket as well as basketball is calculated
by Number divisible by 3 and 5 which will be divisible by the LCM of 3 and
5= 15
Number divisible by 15 from 1 to 100 are 15, 30 , 45, 60 , 75, 90
So, there are total 6 students who will play both cricket and basketball.
36 d Prime factorization of 972 = 22 × 35
Total number of factors of 972 = (2 + 1) (5 + 1) = 3 x 6 = 18
So, Total number of ways that we can express 216 as a product of two of
1
its factors = 2 × 18 = 9
37 a In 2111 × 5222
Number of 2's = 111
Number of 5's = 222
Number of Zeros are found by the pairs of (2×5)
Here Number of 2's are less than number of 5's
Pair (2's & 5's) = 111
∴ Number of zeroes = 11
38 c Let the time spent on each question of Quantitative Ability be 2x minutes.
According to the question,
40 × 2x + 160x = 240 min
⇒ 240x = 240 min
⇒ x = 1 min
Thus, 40 questions of quant section should be attempted in = 40×2x
= 80 minutes
39 b A goes Down 2 steps each time. B comes up 1 step each

A is initially at 22nd step, then they B is initially at 1st step:


move in every step as follows:

20 2
18 3
16 4

www.iassetu.com 20 +91 8826619699


14 5
12 6
10 7
8 8

Hence, they both will meet at 8th


step from bottom.

40 d Let 230 = x. Then, (230+ 1) = (x + 1).


Let (x + 1) be completely divisible by the natural number N. Then,
(290 + 1) = [(230) ^3 + 1] = (x3 + 1) = (x + 1) (x2- x + 1), which is completely
divisible by N, since (x + 1) is divisible by N.
Hence option (d) is the correct answer.
41 d Let a = 4 and b = 2

5a + 3b = 26, which is divisible by 13.

Now, substitute a = 4 and b = 2 in each alternative

Now, 5(a+b) = 5(4 +2) = 5×6 = 30

6ab = 6×4×2 = 48

2(a+3b) = 2×(4+3×2) = 2×14 = 28

4a + 5b = 4×4 + 5×2 = 16 + 10 = 26

Hence, option (d) is correct answer.

42 c Given that Praveen has 42 romantic novels, 78 war stories, and 102
science books in his bookshelf.

So, HCF of 42, 78 and 102 = 6

Minimum number of rows = 42/6 + 78/6 +

102/6 = 7 + 13 + 17 = 37

Hence, option (c) is correct.

43 d Given that p is a prime number and q is a composite number.


So, p can be 2, 3, 5, 7, 11, 13, .….
And q can be 4, 6, 8, 9, 10, .….
Statement 1: p and q can be coprime numbers.
e.g. (3, 4) (7,9) etc. as these sets have HCF 1.

www.iassetu.com 21 +91 8826619699


Thus, statement 1 is correct.
Statement 2: q/p can be a prime number,
e.g. 6/3 = 2. Thus, statement 2 is correct.
Statement 3: p + q can be an odd or an even number,
e.g. 3 + 4 = 7 which is odd and 5+9=14 which is even.
Thus, statement 3 is correct.
Hence all the statements 1, 2 and 3 are correct.
44 a Let the number of pens with A be x and number of pens with B be y.

Case 1:

x + 10 = 3 (y – 10)

⇒ x + 10 = 3y – 30

⇒ x – 3y = -20 ...........(i)

Case 2:

x – 10=y + 10

⇒ x – y = 20. ........... (ii)

From equations (i) and (ii), we get:

x = 40 and y = 20

Hence, option (a) is correct.


45 d As a is an even positive integer, each of the given terms (a + 5) (a + 7) (a +
9) (a + 11) (a + 13) are odd integers and the gap between the consecutive
terms is always 2, the numbers are consecutive odd integers.

Now, in any 5 consecutive odd positive integers, one of them is always


divisible by 3 and one of them is always divisible by 5.

Therefore, their product is always divisible by 3 x 5 = 15

Hence, option (d) is correct answer.

46 d Let the middle digit be x.

The middle digit is half of the sum of the other two digits.
1
Then, x = ×16 or x = 8. So, the number is either 789 or 987.
2

Since the number decreases on reversing the digits, so the hundred's

www.iassetu.com 22 +91 8826619699


digits is larger than the unit's digit.

Hence, required number = 987.

Hence, option (d) is correct answer.

47 d Statement 1: There are 25 prime numbers between 1 to 100 which are 2,


3, 5, 7, 11, 13, 17, 19, 23, 29, 31, 37, 41, 43, 47, 53, 59, 61, 67, 71, 73, 79,
83, 89, 97. Therefore, statement 1 is incorrect.

Statement 2: There are 9 two-digit prime numbers which remain prime


even when the digits are interchanged 11,13,17,31,37,71,73,79 and 97.
Therefore, statement 2 is also incorrect.

Hence, option (d) is correct answer.

48 b Let a three-digit number is xyz.

Sum of the digits = product of the digits.

Only 1, 2 and 3 are three digits where sum of the digits is equal to the
product of its digits.

Possible numbers = 123, 132, 231, 213, 312, 321, hence statement 2 is
correct (number can be either odd or even).

If it is an odd number, then it is not divisible by 2 and 6 and if it is an even


number then it is not divisible by 3. Hence, statement 1 is incorrect.

Hence, option (b) is correct answer.


49 c The first person shook hands with 16 remaining people, the second
person also shook hands with 16 people, but we count 15, as the
handshake with the first person has already been counted.

Then add 14 for the third person, 13 for the fourth one & proceeding in
this fashion.
We get: 16 + 15 + 14 + 13 + 12 + 11 + 10 + 9 + 8 + 7 + 6 + 5 + 4 + 3 + 2 + 1 =
136.
Hence 136 handshakes took place before & 136 after the meeting, For a
total of 272.

www.iassetu.com 23 +91 8826619699


50 b Let the four positive numbers be a, b, c, and d, in that order. Then we
have:
c2 + b2 = a2 + d2 + 4

ac + bd = 11

a + b + c = d + 6,

By Hit and trial method we get the solution as,

a = 3, b = 4, c = 1, d = 2

Hence, option (b) is correct answer.

www.iassetu.com 24 +91 8826619699


Q.NO Answer Solution
51 B Solution:
Let’s assume A = 3x (x is any number)
Same way B = 2x
Possible value of A is even or odd both
So A + B can be Even or Odd. It’s not necessary always divisible by 2 so statement (i) is NOT
CORRECT.
B2 = (2x)2 = 4x2
Here B2 is always divisible by 4. So, statement (ii) is CORRECT.
A × B = 3x × 2x
= 6x2
So, A×B always divisible by 6. Statement (iii) is CORRECT.
Option (B) ii & iii only CORRECT ANSWER.

52 A Solution:
Solve it using Reverse Remainder Theorem (RRT),
91
Let’s 95 here using simple Remainder Theorem value of remainder is 91 but using Reverse
Remainder Theorem value of remainder is (-4).
93 94
Same way remainder is (-2) and remainder is (-1).
95 95
96
And Using Remainder Theorem remainder of ( ) is 1.
95
97 98 99
Same way, remainder of (95) is 2, remainder of (95) is 3, remainder of (95) is 4, remainder of
100
( ) is 5.
95
−4 × −2 × −1 × 1 × 2 × 3 × 4 × 5
Now remainder =
95
−8×120
=
95
−960
=
95
−8×120
= 95
−960
=
95
−960
= remainder of( 95 )
= -10
Here remainder is in negative so subtract from deviser which is 95 – 10 = 85
So remain Remainder is 87 option (A).

53 A Solution:
P23 + PP6 + 26P + PPP = 6PP (expand the expression using its position)
100P + 20 + 3 + 100P +10P + 6 + 200 + 60 + P + 100P + 10P + P = 600 + 10P + P
322P + 289 = 11P + 600
322P – 11P = 600 – 289
311P = 311
P=1
So in given expression put the value,
(𝑃∗𝑃)+ 1 (1∗1)+ 1 2
P =1 put in the expression, = 2 = 2 = 1.
2
Option (A) 1 ANSWER

54 B Solution:
Time saving Technique 6n(n = any natural number) is given the unit digit of 6n is 6.
Answer is OPTION (B) 6

55 A Solution:

www.iassetu.com 25 +91 8826619699


(ax+1)n
Remember Equation =1
𝑎
(15×6 + 1)5324
Write it in equation form so the remainder is 1.
15
Answer is Option (A) 1

56 B Solution:
To find number of ZEROs we have to find pairs of 5 × 2 because zeros = number of pairs of 5 ×
2.
15 × 18 × 20 × 22 × 25 × 95 × 155 × 175 × 32
= (5 × 3) × (3 × 3 × 2) × (5 × 2 × 2) × (2 × 11) × (5 × 5) × (19 × 5) × (31 × 5) × (7 × 5 × 5) × (2
× 2 × 2 × 2 × 2)
= (5 × 2) × (5 × 2) × (5 × 2) × (5 × 2) × (5 × 2) × (5 × 2) × (5 × 2) × (5 × 2) × 2 × 3 × 3 × 3 × 7 ×
11 × 19 × 31
Here 8 pair of 5 × 2 is possible. So number of ZEROs at the last in this expression is 8 option
(B).

57 A Solution:
67
Remainder of 64 is 3.
71 73 83
Same way Remainder of 64 is 7, Remainder of 64 is 9, Remainder of 64 is 19.
67 × 71 × 73 × 83
Remainder of
64
67 71 73 83
Remainder of × Remainder of × Remainder of × Remainder of
64 64 64 64
=
64
Remainder of (3 ×7 ×9 ×19)
=
64
Remainder of (27 ×133)
=
64
27 133
Remainder of × Remainder of
64 64
=
64
27×5
Remainder of
64
=
64
135
Remainder of
64
=
64
7
= 64
So remainder is 7.

58 A Solution:
Given series is Fibonacci series,
Find nth position number = (n-1)th number + (n-2)th number
1st number = 0 + 1 = 1
2nd number = 1 + 1 = 2
3rd number = 1 + 2 = 3
4th number = 2 + 3 = 5
Same way x = 8 + 13 = 21
Answer is Option (A) 21.

59 A Solution:
A = 41p + 21
B = 41q + 16
C = 41r + 32
Lets 3A + 3B – 2C = 3(41p + 21) + 3(41q + 16) – 2(41r + 32)
= 123p + 63 + 123q + 48 – 82r – 64
= 41 × 3(p+q) – 41 × 2 × r + 47
41 × 3(p+q) is divisible by 41, same way 41 × 2 × r is divisible by 41
Only left with 47.

www.iassetu.com 26 +91 8826619699


47
Remainder of is 6.
41
Answer is Option (A) 6

60 A
Solution:
Natural number mean {1,2,3,4,5,...,∞}
Here only single digit number possible which is 1, 2, 3, 4, 5, 6, 7, 8, 9.
So it is clear that sum of 2 digit can neither be more than 18 nor less than 2.
ABCD
+ EFGH
1 2 3 2 1
D + H = 1 not possible so it has to be 11
1 (carry)
ABCD
+ EFGH
1 2 3 2 1
Now C + G + 1 = 12 (possible sum of the 2 digit may be 2 or 12 but here carry is already 1 so
sum has to be 12)
C + G = 11 1 (carry)
ABCD
+ EFGH
1 2 3 2 1
B + F > C + G so B + F > 11 So B + F + 1 = 13
1 (carry)
ABCD
+ EFGH
1 2 3 2 1
Here A + E + 1 = 12 so A + E = 11
Now x = A + C + E + G = (A + E) + (C + G) = 11 + 11 = 22
x+2
( 6 )3
22+2 3
=( )
6
24 3
= ( 6 )
= 43
= 64
Answer is Option (A) 64.

61 A Solution:
Let's take any minimum 4-digit number like 1005
1005
Remainder of 19 is 17 so the minimum 4-digit number has to be 1003 remainder is 15
1003 1003 1003
Now Remainder of 16 is 11, Remainder of is 0, Remainder of is 13.
17 18
Answer is Option (A)

62 C Solution:
Directly eliminate option A & D because option A has one digit 3 but 232 not divisible by 3 so
option A not possible same way Option D has one digit 5 but 534 not divisible by 5 so option D
not Possible
Now, from Option B 342 is divisible by 2 and 3 but not divisible by 4 so option B is eliminated
Hence answer is Option (C) 432.

63 B Solution:
Number of Zeros = number of pairs of 5 × 2.
5 × 9 × 13 × 15 × 18 × 20 × 35 × 48 × 60 × 125 × 144

www.iassetu.com 27 +91 8826619699


= (5) × (3 × 3) × (13) × (5 × 3) × (3 × 3 × 2) × (5 × 2 × 2) × (7 × 5) × (3 × 2 × 2 × 2 × 2) × (2 × 2
× 3 × 5) × (5 × 5 × 5) × (2 × 2 × 2 × 2 × 3 × 3)
=(5 × 2) × (5 × 2) × (5 × 2) × (5 × 2) × (5 × 2) × (5 × 2) × (5 × 2) × 2 × 2 × 2 × 2 × 2 × 2 × 3 × 3
× 3 × 3 × 3 × 3 × 3 × 3 × 3 × 7 × 13
Here 7 pairs of 5 × 2 is possible.
Answer is Option (B) 7.

64 B Solution:
Here 2 possibilities
Possibility -1 A & C is Even.
(A2 + B)(C2 + D) here A & C is even so square of even also even and add odd number gives odd
number. Multiplication of 2 odd number gives Odd number so statement (i) is NOT CORRECT.
(A + B2)(C2 + D) here B is odd and square of odd number gives Odd number. So it gives odd
number then A is even so final answer of 1st bracket is odd and odd multiply with any odd
number it gives Odd number only. So statement (ii) is CORRECT.
(A × C)(B × D) here Even number multiply with Even(A × C) gives Even Number and Odd
multiply with Odd(B × D) gives Odd number. But Odd multiply with Even number gives Even
number. So statement (iii) is correct.
Similarly,
possibility -2 where B & D is even.
Statement (i) NOT CORRECT, Statement (ii) is CORRECT and Statement (iii) is CORRECT.
Answer is Option (B) ii & iii CORRECT.

65 A
Solution:
158902 × 2591ø × 5821
Here
10ø
158902 × 2591 × 5821
= 10
158902 2591 5821
Remainder of 10 is 2, Remainder of 10
is 1 And Remainder of 10
is 1
2×1×1
So, remainder of 10 is 2.
Answer is Option (A) 2.

66 B Solution:
Given Number is 65892a5 divisible by 15 means divisible by 5 × 3 so here we check divisibility
rule of 3 and 5 for every option but 65892a5 last digit is 5 so it is divisible by 5 so no need to
check for 5 now only need to check divisibility rule of 3.
Divisibility rule of 3 is sum of the all digits of the given number has to be divisible by 3.
65892a5 =6+5+8+9+2+a+5
= 35 + a
Let’s choose value of a from the options,
a = 2, =35 + 2 = 37 (Not divisible by 3)
a = 4, = 35 + 4 = 39 (Divisible by 3)
a = 6, = 35 + 6 = 41 (Not divisible by 3)
a = 8, = 35 + 8 = 43 (Not divisible by 3)
Answer is Option (B) 4 and Number is 6589245.
67 A Solution:
A8 + 2A + 31 + A1 = 10A (Expand expression using it’s position)
10A + 8 + 20 + A + 31 + 10A + 1 = 100 + A
21A + 60 = 100 + A
20A = 40
A = 2.
Answer is Option (A) 2.

www.iassetu.com 28 +91 8826619699


68 C
Solution:
A = -, B = +, C = ×, D = ÷
525 − 25 + 50 × 100 ÷ 5 − 60
Replace alphabets by symbols in the expression X = .
125+ 25 ÷ 5 − 50
525 − 25 + 50 × 20 − 60
X= 125+ 5 − 50
500 + 1000 − 60
=
80
1440
= 80
144
=
8
X = 18.
2X + 1 = (2 × 18) + 1
= 36 + 1
= 37
37 is a prime number. So, Statement 1 is Correct.
X2 – 2X + 1 = 172
(18 × 18) – 2(18) + 1 = 289
324 – 36 + 1 = 289
289 = 289
So, Statement 2 is Correct.
Answer is Option (C) Both 1 and 2.

69 C Solution:
Statement 1:
Let the two consecutive odd integers is 2x + 1 and 2x + 3.
(2x + 3)2 − (2x + 1)2 = 4x2 + 12x + 9 – 4x2 − 4x – 1 = 8x + 8 = 8(x + 1)
So, the difference of 2 consecutive odd integers square always divisible by 8.
Statement 1 is correct.
Statement 2: let’s take smallest 4 consecutive even number which is 2, 4, 6, 8.
Now the product of all this 4 number = 2 × 4 × 6 × 8 = 384.
Now take any consecutive 4 numbers like (22,24,26,28) = 22 × 24 × 26 × 28
= 11 × 2 × 4 × 3 × 2 ×13 ×2 ×7 ×2 ×2
= 2 × 4 × 6 × 8 (11 × 13 × 7)
= 384 (77 × 13)
So it is proved that 4 consecutive even number is divisible by 384.
Thus, statement 2 is correct.
Answer is Option (C) Both 1 and 2.

70 B Solution:
Time Saving Technique: (ax-bx) is always divisible by (a-b).
Here a – b = 36 – 33 = 3.
Answer is Option (B) 3.

71 C Solution:
Statement 1: from 0 to 9 only 1 and 9 both are single digit odd and non prime number not only 9.
So statement 1 is NOT CORRECT.
Statement 2: from 0 to 9 only 2 is the single digit even prime number.
So statement 2 is CORRECT.

Prime numbers between 0 and 50 = {2, 3, 5, 7, 11, 13, 17, 19, 23, 29, 31, 37, 41, 43, 47}
Here, Total prime numbers between 0 and 50 are 15.
So, Statement 3 is Not Correct.

Total prime number between 40 and 50 = {41, 43, 47} = 3

www.iassetu.com 29 +91 8826619699


Total prime number between 70 and 80 = {71, 73, 79} = 3
So, Statement 4 is Correct.

In the question they ask about NOT CORRECT option which is statement 1 and 3.
Answer is Option (C) Only 1 and 3.

72 A Solution:
2x2 - 38x + 68 = 0
x2 - 19x + 34 = 0 (divide both side by 2)
x2 - 17x – 2x + 34 = 0
x(x-17)-2(x-17) = 0
(x - 17)(x - 2) = 0
X – 17 = 0 or x - 2 = 0
x = 17 x = 2
Here, possible value of x is 17 and 2 and sum of both is 19.
Answer is Option (A) 19.

73 D

Solution:
Number of Zeros in 101!(Find number of 5s in 101!)

5 101
20
4
24 Total
Number of Zeros in 151!(Find number of 5s in 151!)
5 151
30
6
1
37 Total
Total Zeros in 101! × 151! = Number of zeros in 101! + Number of Zeros in 151!
= 24 + 37
= 61.
Answer is Option (D) 61.

74 B Solution:
Number = 563P32
Divisibility Rule of 6 is if number is divisible by 3 and 2 both then number is divisible by 6.
Here 563P32 is Even number so divisible by 2.
Divisibility rule of 3 is like sum of the all digits has to be divisible by 3.
=5+6+3+P+3+2
=19 + P
Now from the options let’s have P = 3,
= 19 + 3
= 22 (Not divisible by 3 so P = 3 not possible)
Take P = 2,
= 19 + 2
= 21 (now Divisible by 3 so value of P is 2)
6-digit number is 563232.
P=2

www.iassetu.com 30 +91 8826619699


Answer is Option (B) P = 2

75 B Solution: (A) 250


@=+
$=-
#=×
& = (_×_)2
x = (3 × 5)2 + 59 – 17 × 2
= (15)2 + 59 – 34
= 225 + 25
x = 250.
x is divisible by 2, 5 and 10 but not divisible by 20. So, Statement 1 is Not Correct.
x = 250
So, √x = √250
= √25 × 10
= 5√10
5 × 2√10
= 2
(Multiply and Divided by 2)
5√10 × 4
=
2
5√40
√x = 2
So, Statement 2 is Correct.
Answer is Option (B) Only Statement 2.

www.iassetu.com 31 +91 8826619699


www.iassetu.com 32 +91 8826619699

You might also like